Stanford Banner

Algebra Identities

    1. Sum of the first n integers \( 1+2+\cdots+n=\frac{n(n+1)}{2} \)
    2. Sum of squares 1^2+2^2+\cdots+n^2=\frac{n(n+1)(2n+1)}{6}
    3. Sum of cubes 1^3+2^3+\cdots+n^3=\left(\frac{n(n+1)}{2}\right)^2
    4. Finite geometric series 1+r+r^2+\cdots+r^n=\frac{r^{\,n+1}-1}{r-1}
      |r|<1: Infinite geometric: 1+r+r^2+\cdots=\frac{1}{1-r}
    5. Telescoping Series \frac{1}{1\cdot2}+\frac{1}{2\cdot3}+\cdots+\frac{1}{n(n+1)} =\left(1-\frac{1}{2}\right)+\cdots+\left(\frac{1}{n}-\frac{1}{n+1}\right)=1-\frac{1}{n+1}
    6. Square of a sum (a+b)^2=a^2+2ab+b^2
    7. Square of a difference (a-b)^2=a^2-2ab+b^2
    8. Cube of a sum (a+b)^3=a^3+3a^2b+3ab^2+b^3
    9. Cube of a difference (a-b)^3=a^3-3a^2b+3ab^2-b^3
    10. Difference of squares a^2-b^2=(a-b)(a+b)
    11. Difference of cubes a^3-b^3=(a-b)(a^2+ab+b^2)
    12. Sum of cubes (factored) a^3+b^3=(a+b)(a^2-ab+b^2)
    13. General difference of powers a^n-b^n=(a-b)\left(a^{n-1}+a^{n-2}b+\cdots+b^{n-1}\right)
    14. Sum of odd power (n odd) n\ \text{odd}:\ a^n+b^n=(a+b)\left(a^{n-1}-a^{n-2}b+\cdots+b^{n-1}\right)
    15. Difference of even power (n even) n\ \text{even}:\ a^n-b^n=(a+b)\left(a^{n-1}-a^{n-2}b+\cdots-b^{n-1}\right)
    16. Hockey Stick Identity \binom{k}{k}+\binom{k+1}{k}+\binom{k+2}{k}+\cdots+\binom{n}{k}=\binom{n+1}{k+1}
    17. Vandermonde’s Identity \binom{m}{0}\binom{n}{k}+\binom{m}{1}\binom{n}{k-1}+\binom{m}{2}\binom{n}{k-2}+\cdots+\binom{m}{k}\binom{n}{0}=\binom{m+n}{k}

      \binom{n}{0}^2+\binom{n}{1}^2+\binom{n}{2}^2+\cdots+\binom{n}{n}^2=\binom{2n}{n}

    18. Symmetric cubic sum a^3+b^3+c^3=(a+b+c)(a^2+b^2+c^2-ab-ac-bc)+3abc
    19. Factoring quadratic by grouping x^2+(a+b)x+ab=(x+a)(x+b)
    20. Binomial Theorem (a+b)^n=\binom{n}{0}a^n+\binom{n}{1}a^{n-1}b+\binom{n}{2}a^{n-2}b^2+\cdots+\binom{n}{n}b^n
    21. Quadratic Formula ax^2+bx+c=0\ \Rightarrow\ x=\frac{-b\pm\sqrt{b^2-4ac}}{2a}
    22. Approaches to Similar Equations \text{Addition and Subtraction}
    23. SFFT (Simon’s Favorite Factoring Trick) xy + ax + by = c \Rightarrow (x + b)(y + a) = c + ab

      BFFT (Bateni’s Favorite Factoring Trick 😊) axy + bx + cy = d
      a^2xy + abx + acy = ad
      (ax + c)(ay + b) = ad + bc

    24. Square of a sum (n terms) (x_1 + x_2 + \cdots + x_n)^2 = x_1^2 + x_2^2 + \cdots + x_n^2 + 2x_1x_2 + 2x_1x_3 + \cdots + 2x_{n-1}x_n
    25. Arithmetic Sequence and Series a_n = a_1 + (n-1)d
      S_n = a_1 + a_2 + \cdots + a_n = \frac{n}{2}(a_1 + a_n)
    26. Expansion (or factorization) of three binomials abc-ab-ac-bc+a+b+c-1=(a-1)(b-1)(c-1)
    27. Expansion (or factorization) of two binomials a^2 + ab + ac + bc = (a + b)(a + c)
    28. Max–QM–AM–GM–HM–Min Inequality x_1,\ldots,x_n \in \mathbb{R}^+
      \max\{x_i\} \;\ge\; \sqrt{\frac{x_1^2+x_2^2+\cdots+x_n^2}{n}} \;\ge\; \frac{x_1+x_2+\cdots+x_n}{n} \;\ge\; \sqrt[n]{x_1x_2\cdots x_n} \;\ge\; \frac{n}{\frac{1}{x_1}+\frac{1}{x_2}+\cdots+\frac{1}{x_n}} \;\ge\; \min\{x_i\}
      Equality holds if and only if all x_i are equal.
    29. Integer and Fractional Parts
      \lfloor x \rfloor: Integer part    \{x\}: Fractional part

      We can write x as:
      x = k + r,\qquad k \in \mathbb{Z},\ r \in \mathbb{R},\ 0 \le r < 1

    30. Vieta’s Formula For a quadratic:
      ax^2+bx+c=0 (roots: r_1,r_2)
      r_1+r_2=-\frac{b}{a}, \qquad r_1r_2=\frac{c}{a}

      For a cubic:
      ax^3+bx^2+cx+d=0 (roots: r_1,r_2,r_3)
      r_1+r_2+r_3=-\frac{b}{a}, \qquad r_1r_2+r_1r_3+r_2r_3=\frac{c}{a}, \qquad r_1r_2r_3=-\frac{d}{a}

      For a general polynomial:
      a_nx^n+a_{n-1}x^{\,n-1}+a_{n-2}x^{\,n-2}+\cdots+a_1x+a_0=0 (roots: r_1,\ldots,r_n)
      \begin{aligned} r_1+r_2+\cdots+r_n &= -\frac{a_{n-1}}{a_n},\\ \sum_{1\le i<j\le n} r_ir_j &= \frac{a_{n-2}}{a_n},\\ \sum_{1\le i<j<k\le n} r_ir_jr_k &= -\frac{a_{n-3}}{a_n},\\ &\ \vdots\\ r_1r_2\cdots r_{n-1} &= (-1)^{n-1}\frac{a_1}{a_n},\\ r_1r_2\cdots r_n &= (-1)^{n}\frac{a_0}{a_n} \end{aligned}

    31. Signs and Roots of Quadratics
      Sign and the Number of Roots of a Quadratic Function
    32. Remainder of Polynomial Division \begin{aligned} A(x) \div B(x):\quad A(x) = B(x)Q(x) + R(x),\quad \deg(R(x)) < \deg(B(x)) \\ \Rightarrow\ \text{If } B(x) = 0,\ \text{ then } A(x) = R(x) \end{aligned}
    33. Number of Positive Divisors

      If

      n = p_1^{\alpha_1} p_2^{\alpha_2} \cdots p_k^{\alpha_k},

      then the number of positive divisors of n is

      (\alpha_1 + 1)(\alpha_2 + 1)\cdots(\alpha_k + 1).

    34. Sum of Positive Divisors

      If

      n = p_1^{\alpha_1} p_2^{\alpha_2} \cdots p_k^{\alpha_k},

      then the sum of the positive divisors of n is

      (1 + p_1 + p_1^2 + \cdots + p_1^{\alpha_1})(1 + p_2 + p_2^2 + \cdots + p_2^{\alpha_2}) \cdots (1 + p_k + p_k^2 + \cdots + p_k^{\alpha_k}).

      which can also be written as

      \frac{p_1^{\alpha_1 + 1} - 1}{p_1 - 1} \times \frac{p_2^{\alpha_2 + 1} - 1}{p_2 - 1} \times \cdots \times \frac{p_k^{\alpha_k + 1} - 1}{p_k - 1}.

    35. Product of Positive Divisors

      If

      n = p_1^{\alpha_1} p_2^{\alpha_2} \cdots p_k^{\alpha_k},

      let

      t = (\alpha_1 + 1)(\alpha_2 + 1)\cdots(\alpha_k + 1).

      Then the product of the positive divisors of n is

      n^{t/2}.

    36. General Solutions of Trigonometric Equations

      \sin \alpha = \sin \beta \Rightarrow \alpha = \beta + 360k,\ 180 - \beta + 360k,\ k \in \mathbb{Z}

      \cos \alpha = \cos \beta \Rightarrow \alpha = \pm \beta + 360k,\ k \in \mathbb{Z}

      \tan \alpha = \tan \beta \Rightarrow \alpha = \beta + 180k,\ k \in \mathbb{Z}

      \cot \alpha = \cot \beta \Rightarrow \alpha = \beta + 180k,\ k \in \mathbb{Z}

    37. Trigonometric Transformations

      \sin(90^\circ - \alpha) = \cos \alpha

      \cos(90^\circ - \alpha) = \sin \alpha

      \tan(90^\circ - \alpha) = \cot \alpha

      \cot(90^\circ - \alpha) = \tan \alpha

       

      \sin(180^\circ - \alpha) = \sin \alpha

      \cos(180^\circ - \alpha) = -\cos \alpha

      \tan(180^\circ - \alpha) = -\tan \alpha

      \cot(180^\circ - \alpha) = -\cot \alpha

       

      \sin(-\alpha) = -\sin \alpha

      \cos(-\alpha) = \cos \alpha

      \tan(-\alpha) = -\tan \alpha

      \cot(-\alpha) = -\cot \alpha

      \sin(90^\circ + \alpha) = \cos \alpha

      \cos(90^\circ + \alpha) = -\sin \alpha

      \tan(90^\circ + \alpha) = -\cot \alpha

      \cot(90^\circ + \alpha) = -\tan \alpha

       

      \sin(180^\circ + \alpha) = -\sin \alpha

      \cos(180^\circ + \alpha) = -\cos \alpha

      \tan(180^\circ + \alpha) = \tan \alpha

      \cot(180^\circ + \alpha) = \cot \alpha

    38. Trigonometric Identities

      \sin^2 \alpha + \cos^2 \alpha = 1

      \sin(\alpha + \beta) = \sin \alpha \cos \beta + \cos \alpha \sin \beta

      \cos(\alpha + \beta) = \cos \alpha \cos \beta - \sin \alpha \sin \beta

      \sin 2\alpha = 2 \sin \alpha \cos \alpha

      \cos 2\alpha = \cos^2 \alpha - \sin^2 \alpha = 2\cos^2 \alpha - 1 = 1 - 2\sin^2 \alpha

      \cos 3\alpha = 4\cos^3 \alpha - 3\cos \alpha

      \tan(\alpha + \beta) = \frac{\tan \alpha + \tan \beta}{1 - \tan \alpha \tan \beta}

      \cot(\alpha + \beta) = \frac{\cot \alpha \cot \beta - 1}{\cot \alpha + \cot \beta}

      \sin x + \cos x = \sqrt{2} \sin(x + 45^\circ)

      \sin x - \cos x = \sqrt{2} \sin(x - 45^\circ)

    39. Trigonometric Sum-to-Product Identities

      \sin \alpha + \sin \beta = 2 \sin \left( \frac{\alpha + \beta}{2} \right) \cos \left( \frac{\alpha - \beta}{2} \right)

      \sin \alpha - \sin \beta = 2 \cos \left( \frac{\alpha + \beta}{2} \right) \sin \left( \frac{\alpha - \beta}{2} \right)

      \cos \alpha + \cos \beta = 2 \cos \left( \frac{\alpha + \beta}{2} \right) \cos \left( \frac{\alpha - \beta}{2} \right)

      \cos \alpha - \cos \beta = -2 \sin \left( \frac{\alpha + \beta}{2} \right) \sin \left( \frac{\alpha - \beta}{2} \right)

    40. Trigonometric Product-to-Sum Identities

      \sin p \cos q = \frac{1}{2} \left( \sin(p + q) + \sin(p - q) \right)

      \cos p \cos q = \frac{1}{2} \left( \cos(p + q) + \cos(p - q) \right)

      \sin p \sin q = \frac{1}{2} \left( \cos(p - q) - \cos(p + q) \right)



AMC 10A — Problems

AMC 10A 2024 ( Problem 11 )

How many ordered pairs of integers (m,n) satisfy \sqrt{n^2 - 49} = m ?

  • 1
  • 2
  • 3
  • 4
  • infinitely many

Hints & Final Answer
Hint 1

Square both sides.

Hint 2

Rearrange and use Identity 10 (Difference of Squares).

Hint 3

n^2-m^2=(n-m)(n+m)=49

Hint 4

n-m and n+m are factors of 49.

Final Answer

(B) 2

AMC 10/12A 2022 ( Problem 16 )

The roots of the polynomial 10x^3 - 39x^2 + 29x - 6 are the height, length, and width of a rectangular box (right rectangular prism). A new rectangular box is formed by lengthening each edge of the original box by 2 units. What is the volume of the new box?

  • \dfrac{24}{5}
  • \dfrac{42}{5}
  • \dfrac{81}{5}
  • 30
  • 48
Hints & Final Answer
Hint 1

Use 30. Vieta’s Formula.

Hint 2

\text{Volume} = (r_1 + 2)(r_2 + 2)(r_3 + 2)

Hint 3

By using 30. Vieta’s Formula:
\begin{aligned} r_1 + r_2 + r_3 &= \frac{39}{10}, \\ r_1r_2 + r_1r_3 + r_2r_3 &= \frac{29}{10}, \\ r_1r_2r_3 &= \frac{6}{10}. \end{aligned}

Hint 4

Expand:
(r_1 + 2)(r_2 + 2)(r_3 + 2) = r_1r_2r_3 + 2(r_1r_2 + r_2r_3 + r_3r_1) + 4(r_1 + r_2 + r_3) + 8. Substitute from Vieta’s results:
\frac{6}{10} + 2\left(\frac{29}{10}\right) + 4\left(\frac{39}{10}\right) + 8 = 30.

Final Answer

(D) 30

AMC 10/12 A Fall 2021 ( Problem 17 )

For how many ordered pairs \((b,c)\) of positive integers does neither \(x^2+bx+c=0\) nor \(x^2+cx+b=0\) have two distinct real solutions?

  • 4
  • 6
  • 8
  • 12
  • 16
Hints & Final Answer
Hint 1

Use 31. Signs and Roots of Quadratics.

Hint 2

\[ \Delta \le 0 \Rightarrow b^2-4c \le 0 \Rightarrow b^2 \le 4c, \quad c^2-4b \le 0 \Rightarrow c^2 \le 4b. \]

Hint 3

From above, \[ b^2 \le 4c \text{ and } c^2 \le 4b \Rightarrow b^4 \le 16c^2 \le 64b \Rightarrow b^3 \le 64 \Rightarrow b \le 4. \] Similarly \(c \le 4\).

Hint 6

Case \(b=3\): \(9 \le 4c \Rightarrow c=3,\) and \(c^2 \le 12\) holds.

Hint 7

Case \(b=4\): \(16 \le 4c \Rightarrow c=4,\) and \(c^2 \le 16\) holds. Checking all valid \((b,c)\) up to 4 gives 6 pairs.

Final Answer: (B) 6

AMC 10/12A Spring 2021 ( Problem 10 )

Which of the following is equivalent to \[ (2+3)(2^2+3^2)(2^4+3^4)(2^8+3^8)(2^{16}+3^{16})(2^{32}+3^{32})(2^{64}+3^{64})\ ? \]

  • \(3^{127}+2^{127}\)
  • \(3^{127}+2^{127}+2\cdot3^{63}+3\cdot2^{63}\)
  • \(3^{128}-2^{128}\)
  • \(3^{128}+2^{128}\)
  • \(5^{127}\)
Hints & Final Answer
Hint 1

Use 10. Difference of squares.

Hint 2

Multiply by \((3-2)\).

Hint 3

\[ (3-2)(2+3)(2^2+3^2)(2^4+3^4)\cdots(2^{64}+3^{64}) =(3^2-2^2)(2^2+3^2)(2^4+3^4)\cdots \] \[ =(3^4-2^4)(2^4+3^4)\cdots =(3^8-2^8)(2^8+3^8)\cdots \Rightarrow \cdots = 3^{128}-2^{128}. \] So the original product is \((3^{128}-2^{128})/(3-2)=3^{128}-2^{128}.\)

Final Answer

(C) \(3^{128}-2^{128}\)



AMC 10/12A Spring 2021 ( Problem 14 )

All the roots of the polynomial \(z^6-10z^5+Az^4+Bz^3+Cz^2+Dz+16\) are positive integers, possibly repeated. What is the value of \(B\)?

  • \(-88\)
  • \(-80\)
  • \(-64\)
  • \(-41\)
  • \(-40\)
Hints & Final Answer
Hint 1

Use 30. Vieta’s Formula.

Hint 2

Sum of roots: \(r_1+\cdots+r_6=10.\) Product of roots: \(r_1r_2r_3r_4r_5r_6=16.\)

Hint 3

The only way with positive integers: roots \(2,2,2,2,1,1\).

Hint 4

By Vieta, the coefficient of \(z^3\) is \(-\!\sum r_ir_jr_k = -B\).

Hint 5

Count distinct triple products: \[ \binom{4}{3}(2\cdot2\cdot2) +\binom{4}{2}\binom{2}{1}(2\cdot2\cdot1) +\binom{4}{1}\binom{2}{2}(2\cdot1\cdot1) =32+48+8=88. \] So \(B=-88\).

Final Answer

(A) \(-88\)

AMC 10/12 A 2020 ( Problem 21 )

There exists a unique strictly increasing sequence of nonnegative integers \(a_1 < a_2 < \dots < a_k\) such that \[ \frac{2^{289} + 1}{2^{17} + 1} = 2^{a_1} + 2^{a_2} + \dots + 2^{a_k}. \] What is \(k\)?

  • 117
  • 136
  • 137
  • 273
  • 306
Hints & Final Answer
Hint 1

Use the identity for the sum of odd powers (when the exponent is odd): \[ u^{2n+1} + 1 = (u+1)\big(u^{2n} – u^{2n-1} + u^{2n-2} – \cdots – u + 1\big). \] Here \(u = 2^{17}\) and \(2n+1 = 17\cdot17 = 289\) is odd.

Hint 2

Factor the numerator: \[ 2^{289} + 1 = \big(2^{17}\big)^{17} + 1 = \big(2^{17} + 1\big) \Big( (2^{17})^{16} – (2^{17})^{15} + (2^{17})^{14} – \cdots + (2^{17})^{2} – (2^{17})^{1} + 1 \Big). \] Therefore, \[ \frac{2^{289}+1}{2^{17}+1} = (2^{17})^{16} – (2^{17})^{15} + (2^{17})^{14} – \cdots + (2^{17})^{2} – (2^{17})^{1} + 1. \] This is an alternating sum of 17 terms, starting with positive \((2^{17})^{16}\) and ending with \(+1\).

Hint 3

Key fact we’ll reuse: \[ 2^n + 2^n = 2^{n+1}. \] This lets us rewrite differences like \((2^{17})^m – (2^{17})^{m-1}\) as a sum of distinct powers of \(2\) with consecutive exponents.

Hint 4

Look at one “block”: \[ (2^{17})^{2} – (2^{17})^{1} = 2^{34} – 2^{17}. \] Now expand the difference of powers in base 2: \[ 2^{34} – 2^{17} = 2^{34} + 2^{33} – 2^{33} – 2^{17} = 2^{34} + 2^{33} + \cdots + 2^{18} + 2^{17} – 2^{17}. \] More cleanly (using geometric series structure): \[ 2^{34} – 2^{17} = 2^{17}\big(2^{17}-1\big) = 2^{33} + 2^{32} + 2^{31} + \cdots + 2^{18} + 2^{17} – 2^{17} = 2^{33} + 2^{32} + \cdots + 2^{18}. \] This is a sum of 16 consecutive powers: \(2^{18}, 2^{19}, \ldots, 2^{33}.\) Including also that final lone \(+2^{17}\) from elsewhere will give us 17 distinct powers in total for that pair. In words: each adjacent pair \((2^{17})^{m} – (2^{17})^{m-1}\) “unpacks” into 17 distinct powers of 2 with consecutive exponents.

Hint 5

Similarly, for higher exponents: \[ (2^{17})^{4} – (2^{17})^{3} = 2^{68} – 2^{51} = 2^{67} + 2^{66} + \cdots + 2^{52} + 2^{51}. \] Again we get 17 distinct consecutive powers of 2.

Hint 6

The whole expression \[ (2^{17})^{16} – (2^{17})^{15} + (2^{17})^{14} – \cdots + (2^{17})^{2} – (2^{17})^{1} + 1 \] can be grouped into 8 “difference blocks” of the form \((2^{17})^{m} – (2^{17})^{m-1}\), each block expanding into 17 distinct powers of \(2\) with unique exponents, plus the final \(+1 = 2^0\). Because the exponents in different blocks don’t overlap, the binary expansion of the entire sum is just a bunch of distinct powers of 2, no carries needed. So we get: – one term from the final \(+1\), – plus \(8\) blocks, – with \(17\) distinct powers of \(2\) contributed per block.

Hint 7

Therefore \[ k = 1 + 17 \times 8 = 1 + 136 = 137. \]

Final Answer

(C) 137

AMC 10A 2020 ( Problem 14 )

Real numbers x and y satisfy x + y = 4 and x \cdot y = -2. What is the value of x + \dfrac{x^3}{y^2} + \dfrac{y^3}{x^2} + y?

  • 360
  • 400
  • 420
  • 440
  • 480
Hints & Final Answer
Hint 1

Calculate x^2 + y^2.

Hint 2

By using 6. Square of a Sum: x^2 + y^2 = (x + y)^2 - 2xy = 4^2 - 2(-2) = 20

Hint 3

Calculate x^3 + y^3.

Hint 4

By using 12. Sum of cubes: x^3 + y^3 = (x + y)(x^2 - xy + y^2) = 4(20 + 2) = 88

Hint 5

Calculate x^5 + y^5.

Hint 6

x^5 + y^5 = (x^2 + y^2)(x^3 + y^3) - (x^2y^3 + x^3y^2) But x^2y^3 + x^3y^2 = x^2y^2(x + y). Since xy = -2 \Rightarrow x^2y^2 = 4, and x + y = 4, we have x^2y^2(x + y) = 4 \cdot 4 = 16. x^5 + y^5 = (20)(88) - 16 = 1760 - 16 = 1744

Hint 7

We want x + \dfrac{x^3}{y^2} + \dfrac{y^3}{x^2} + y = x + y + \dfrac{x^5 + y^5}{x^2y^2} x + y = 4, x^5 + y^5 = 1744, x^2y^2 = 4. \dfrac{x^5 + y^5}{x^2y^2} = \dfrac{1744}{4} = 436 Therefore, x + \dfrac{x^3}{y^2} + \dfrac{y^3}{x^2} + y = 4 + 436 = 440

Final Answer

(D) 440

AMC 10A 2020 ( Problem 8 )

What is the value of 1 + 2 + 3 - 4 + 5 + 6 + 7 - 8 + \cdots + 197 + 198 + 199 - 200 ?

  • 9,800
  • 9,900
  • 10,000
  • 10,100
  • 10,200
Hints & Final Answer
Hint 1

Use 1. Sum of the first n integers.

Hint 2

Let S = 1 + 2 + 3 - 4 + 5 + 6 + 7 - 8 + \cdots + 197 + 198 + 199 - 200 We can rewrite it as: S = (1 + 2 + 3 + \cdots + 200) - 2(4 + 8 + \cdots + 200)

Hint 3

The sum of the first 200 integers: 1 + 2 + \cdots + 200 = \dfrac{200 \times 201}{2} = 20100 The second term is an arithmetic series of 4’s multiples: 4 + 8 + \cdots + 200 = 4(1 + 2 + \cdots + 50) = 4 \left( \dfrac{50 \times 51}{2} \right) = 4(1275) = 5100 Therefore, S = 20100 - 2(5100) = 20100 - 10200 = 9900

Final Answer

(B) 9900

AMC 10A 2020 ( Problem 5 )

What is the sum of all real numbers x for which |x^2-12x+34|=2?

  • 12
  • 15
  • 18
  • 21
  • 25
Hints & Final Answer
Hint 1

x^2-12x+34=\pm2

Hint 2

Case 1: x^2-12x+34=2\Rightarrow x^2-12x+32=0

By using 31. Signs and Roots of Quadratics:
\Delta=12^2-4(32)=144-128=16>0 → there are 2 different real roots r_1,r_2.

By using 30. Vieta’s Formula:
r_1+r_2=-\frac{-12}{1}=12

Hint 3

Case 2: x^2-12x+34=-2\Rightarrow x^2-12x+36=0
(x-6)^2=0 → one root x=6.

The total sum of all real solutions is 12+6=18.

Final Answer

(C) 18

AMC 10A 2012 ( Problem 22 )

The sum of the first m positive odd integers is 212 more than the sum of the first n positive even integers. What is the sum of all possible values of n?

  • 255
  • 256
  • 257
  • 258
  • 259
Hints & Final Answer
Hint 1

1+3+\cdots+(2m-1)=212+\big(2+4+\cdots+2n\big)

Hint 2

Use 25. Arithmetic Sequence and Series.

Hint 3

m^2 = 212 + n(n+1)

Hint 4

Use 6. Square of a Sum and 10. Difference of Squares.

Hint 5

m^2 = 212 + \Big(n+\tfrac12\Big)^2 - \tfrac14

Hint 6

4m^2 = 848 + (2n+1)^2 - 1

Hint 7

4m^2 - (2n+1)^2 = 847

Hint 8

(2m+2n+1)(2m-2n-1)=847

Hint 9

Since 2m+2n+1 and 2m-2n-1 are factors of 847.

Final Answer

(A) 255


AMC 10B — Problems

AMC 10B 2023 ( Problem 22 )

How many distinct values of x satisfy \lfloor x \rfloor^2 - 3x + 2 = 0, where \lfloor x \rfloor denotes the greatest integer less than or equal to x?

  • an infinite number
  • 4
  • 2
  • 3
  • 0
Hints & Final Answer
Hint 1

Use 29. Integer and Fractional Parts.

Hint 2

Let x=k+r with k\in\mathbb{Z}, 0\le r<1. Then \lfloor x\rfloor=k.

Hint 3

Substitute:
k^2-3(k+r)+2=0\ \Rightarrow\ r=\dfrac{k^2-3k+2}{3}

Hint 4

Since 0\le r<1,
0\le \dfrac{k^2-3k+2}{3} < 1.

Hint 5

From the left inequality:
k^2-3k+2\ge 0 \Rightarrow (k-1)(k-2)\ge 0.
Using 31. Signs and Roots of Quadratics: k\in(-\infty,1]\cup[2,\infty).
Since k is an integer \Rightarrow k can be any integer.

Hint 6

From the right inequality:
\dfrac{k^2-3k+2}{3} < 1 \Rightarrow k^2-3k-1<0.
Use 21. Quadratic Formula: roots \dfrac{3\pm\sqrt{13}}{2}\approx-0....,\ 3.... .
By 31. Signs and Roots: -0....\lt k\lt 3....\Rightarrow k\in\{0,1,2,3\}.

Hint 7

Compute r=\dfrac{k^2-3k+2}{3} for each k:
k=0\Rightarrow r=\tfrac{2}{3}; k=1\Rightarrow r=0; k=2\Rightarrow r=0; k=3\Rightarrow r=\tfrac{2}{3}.
Thus x=k+r\in\left\{\tfrac{2}{3},\,1,\,2,\,\tfrac{11}{3}\right\} — four distinct values.

Final Answer

(B) 4

AMC 10B 2023 ( Problem 14 )

How many ordered pairs of integers (m,n) satisfy m^2+mn+n^2=m^2n^2?

  • 7
  • 1
  • 3
  • 6
  • 5
Hints & Final Answer
Hint 1

Add mn to both sides.

Hint 2

Use 6. Square of a sum.

Hint 3

(m+n)^2=m^2n^2+mn

Hint 4

By using 6. Square of a sum:
m^2n^2+mn=\left(mn+\tfrac12\right)^2-\tfrac14

Hint 5

From Hint 3 and Hint 4:
(m+n)^2=\left(mn+\tfrac12\right)^2-\tfrac14

Multiply both sides by 4:
(2m+2n)^2=(2mn+1)^2-1
(2mn+1)^2-(2m+2n)^2=1

By using 10. Difference of Squares:
\big(2mn+1+2m+2n\big)\big(2mn+1-2m-2n\big)=1

Hint 6

Case 1:
\begin{aligned} 2mn+1+2m+2n&=1\\ 2mn+1-2m-2n&=1 \end{aligned}
Adding: 4mn+2=2\Rightarrow mn=0\Rightarrow m=0\ \text{or}\ n=0
If m=0 \Rightarrow 2mn+1+2m+2n=1+2n=1\Rightarrow n=0
\Rightarrow (m,n)=(0,0)

Hint 7

Case 2:
\begin{aligned} 2mn+1+2m+2n&=-1\\ 2mn+1-2m-2n&=-1 \end{aligned}
\Rightarrow 4mn+2=-2\Rightarrow 4mn=-4\Rightarrow mn=-1
If m=1,\ n=-1 then 2mn+1+2m+2n=-2+1+2-2=-1 and 2mn+1-2m-2n=-2+1-2+2=-1.
Similarly, (m,n)=(-1,1).
Solutions: (m,n)=(1,-1),\,(-1,1).

Final Answer

(C) 3

AMC 10B 2022 ( Problem 17 )

One of the following numbers is not divisible by any prime number less than 10. Which is it?

  • \(2^{606}-1\)
  • \(2^{606}+1\)
  • \(2^{607}-1\)
  • \(2^{607}+1\)
  • \(2^{607}+3^{607}\)
Hints & Final Answer
Hint 1

Refer to 13. General difference of powers and 14. Sum of odd powers.

Hint 2

\(a-b \mid a^n-b^n,\quad n\in\mathbb{Z}^+\)

Hint 3

\(a+b \mid a^n+b^n,\quad n \text{ odd}\)

Hint 4

Choice A: \[ 2^{606}-1=(2^2)^{303}-1=4^{303}-1, \] so \(4-1=3 \mid 4^{303}-1\).

Hint 5

Choice B: \[ 2^{606}+1=4^{303}+1, \] so \(4+1=5 \mid 4^{303}+1\).

Hint 6

Choice D: \(2+1=3 \mid 2^{607}+1\).

Hint 7

Choice E: \(2+3=5 \mid 2^{607}+3^{607}\).

Final Answer

(C) \(2^{607}-1\)

AMC 10B 2022 ( Problem 15 )

Let \(S_n\) be the sum of the first \(n\) terms of an arithmetic sequence that has a common difference of \(2\). The quotient \(\dfrac{S_{3n}}{S_n}\) does not depend on \(n\). What is \(S_{20}\)?

  • 340
  • 360
  • 380
  • 400
  • 420
Hints & Final Answer
Hint 1

Use 25. Arithmetic Sequence and Series.

Hint 2

\[ a_n = a_1 + (n-1)d = a_1 + 2(n-1) \]

Hint 3

\[ S_n = \frac{n}{2}(a_1+a_n) = \frac{n}{2}\big(2a_1+2(n-1)\big) \]

Hint 4

\[ S_{3n} = \frac{3n}{2}\big(2a_1+2(3n-1)\big) \]

Hint 5

Let \(a=a_1\). \[ \frac{S_{3n}}{S_n} = \frac{3\big(2a+2(3n-1)\big)}{2a+2(n-1)} \]

Hint 6

Since the ratio is independent of \(n\), \(\dfrac{S_3}{S_1}=\dfrac{S_6}{S_2}.\)

Hint 7

\[ \frac{3(2a+4)}{2a} = \frac{3(2a+10)}{2a+2} \]

Hint 8

\[ \frac{a+2}{a} = \frac{a+5}{a+1} \]

Hint 9

\[ a^2+5a = a^2+3a+2 \Rightarrow 2a=2 \Rightarrow a=1 \]

Hint 10

\[ S_{20} = \frac{20}{2}(a_1+a_{20}) =10\Big(1 + \big(1+19\cdot2\big)\Big) =400 \]

Final Answer

(D) 400

AMC 10B 2022 ( Problem 9 )

The sum

\[ \frac{1}{2!} + \frac{2}{3!} + \frac{3}{4!} + \cdots + \frac{2021}{2022!} \]

can be expressed as \(a – \frac{1}{b!}\), where \(a\) and \(b\) are positive integers. What is \(a+b\)?

  • 2020
  • 2021
  • 2022
  • 2023
  • 2024
Hints & Final Answer
Hint 1

Use 5. Telescoping Series.

Hint 2

\[ \frac{k}{(k+1)!} = \frac{1}{k!} – \frac{1}{(k+1)!} \]

Hint 3

\[ \frac{1}{2!}+\frac{2}{3!}+\cdots+\frac{2021}{2022!} = \left(\frac{1}{1!}-\frac{1}{2!}\right) +\left(\frac{1}{2!}-\frac{1}{3!}\right) +\cdots +\left(\frac{1}{2021!}-\frac{1}{2022!}\right) = \frac{1}{1!}-\frac{1}{2022!} =1-\frac{1}{2022!} \] Therefore \(a=1\) and \(b=2022\), so \(a+b=2023\).

Final Answer

(D) 2023

AMC 10B 2022 ( Problem 21 )

Let \(P(x)\) be a polynomial with rational coefficients such that when \(P(x)\) is divided by \(x^2 + x + 1\), the remainder is \(x + 2\), and when \(P(x)\) is divided by \(x^2 + 1\), the remainder is \(2x + 1\). There is a unique polynomial of least degree with these two properties. What is the sum of the squares of the coefficients of that polynomial?

  • 10
  • 13
  • 19
  • 20
  • 23
Hints & Final Answer
Hint 1

Try small degrees for \(P(x)\). Use Remainder Theorem for quadratics.

Hint 2

If \(\deg P(x)=0\): \(P(x)=c\). Remainder on division by \(x^2+x+1\) would be \(c\), but must be \(x+2\). Impossible.

Hint 3

If \(\deg P(x)=1\): \(P(x)=ax+b\). From \(x^2+x+1\), remainder \(ax+b=x+2\Rightarrow a=1,b=2\). From \(x^2+1\), remainder \(ax+b=2x+1\Rightarrow a=2,b=1\). Contradiction.

Hint 4

If \(\deg P(x)=2\): \(P(x)=ax^2+bx+c\). Mod \(x^2+x+1=0\Rightarrow x^2=-x-1\): \[ ax^2+bx+c = a(-x-1)+bx+c = x(b-a)+(c-a). \] We need \(x(b-a)+(c-a)=x+2\Rightarrow b-a=1,\ c-a=2.\)
Mod \(x^2+1=0\Rightarrow x^2=-1\): \[ ax^2+bx+c = -a+bx+c = bx+(c-a). \] We need \(bx+(c-a)=2x+1\Rightarrow b=2,\ c-a=1.\) But \(c-a\) can’t be both 2 and 1. Impossible.

Hint 5

If \(\deg P(x)=3\): \(P(x)=ax^3+bx^2+cx+d\). In \(x^2+x+1=0\), we have \(x^2=-x-1\) and \(x^3=-x^2-x=1\). Then \[ P(x)\equiv a + b(-x-1) + cx + d = x(c-b) + (a-b+d). \] We want this to be \(x+2\), so \[ c-b=1,\quad a-b+d=2. \] In \(x^2+1=0\), we have \(x^2=-1,\ x^3=-x\). Then \[ P(x)\equiv a(-x)+b(-1)+cx+d = x(c-a)+(d-b). \] We want this \(=2x+1\), so \[ c-a=2,\quad d-b=1. \]

Hint 6

From the system \[ \begin{cases} c-b=1\\ a-b+d=2\\ c-a=2\\ d-b=1 \end{cases} \] Use \(d=b+1\) and \(c=a+2\). Then \(c-b=a+2-b=1 \Rightarrow a-b=-1.\) Also \(a-b+d = a-b+b+1 = 2 \Rightarrow a=1.\) Then \(b=a+1=2,\ c=a+2=3,\ d=3.\)

Final Answer

\(P(x)=x^3+2x^2+3x+3.\) Sum of squares of coefficients: \[ 1^2+2^2+3^2+3^2 = 23. \] (E) 23

AMC 10B Fall 2021 ( Problem 22 )

For each integer \(n \ge 2\), let \(S_n\) be the sum of all products \(jk\), where \(j\) and \(k\) are integers and \(1 \le j < k \le n\). What is the sum of the 10 least values of \(n\) such that \(S_n\) is divisible by 3?

  • 196
  • 197
  • 198
  • 199
  • 200
Hints & Final Answer
Hint 1

Use 1. Sum of the first \(n\) integers and 24. Square of a sum (n terms).

Hint 2

\[ (1+2+\cdots+n)^2 = 1^2+2^2+\cdots+n^2 + 2\big(1\cdot2+1\cdot3+\cdots+(n-1)n\big) \]

Hint 3

\[ S_n = 1\cdot2 + 1\cdot3 + \cdots + (n-1)n = \frac{(1+2+\cdots+n)^2 – (1^2+2^2+\cdots+n^2)}{2} \]

Hint 4

Using known sums: \[ S_n = \frac{\left(\frac{n(n+1)}{2}\right)^2 – \frac{n(n+1)(2n+1)}{6}}{2} = \frac{n(n+1)(3n^2-n-2)}{24} = \frac{n(n+1)(3n+2)(n-1)}{24}. \]

Hint 5

For \(3\mid S_n\) we need \(n(n+1)(3n+2)(n-1)\) to be a multiple of \(9\). Since \(3n+2\) is not a multiple of 3, \((n-1)n(n+1)\) must be a multiple of 9. Among three consecutive integers exactly one is a multiple of 3, so that one must be a multiple of 9. Hence \(n \equiv 1,0,-1 \pmod{9}.\)

Hint 6

Since \(n\ge2\), the 10 least such values are \(8,9,10,17,18,19,26,27,28,35\).

Final Answer

(B) 197

AMC 10B Spring 2021 ( Problem 15 )

The real number \(x\) satisfies the equation \[ x + \frac{1}{x} = \sqrt{5}. \] What is the value of \[ x^{11} – 7x^{7} + x^{3}\, ? \]

  • \(-1\)
  • 0
  • 1
  • 2
  • \(\sqrt{5}\)
Hints & Final Answer
Hint 1

Factor the expression: \[ x^{11} – 7x^{7} + x^{3} = x^{7}\!\Big(x^{4} – 7 + \frac{1}{x^{4}}\Big). \]

Hint 2

We’ll try to compute \(x^{4} + \dfrac{1}{x^{4}}\). To get that, first find \(x^{2} + \dfrac{1}{x^{2}}\). Use the square of a sum.

Hint 3

Calculate \(x^{2} + \dfrac{1}{x^{2}}\).

Hint 4

\[ x^{2} + \frac{1}{x^{2}} = \left(x + \frac{1}{x}\right)^{2} – 2 = (\sqrt{5})^{2} – 2 = 5 – 2 = 3. \]

Hint 5

Now calculate \(x^{4} + \dfrac{1}{x^{4}}\).

Hint 6

Square the result from Hint 4: \[ x^{4} + \frac{1}{x^{4}} = \left(x^{2} + \frac{1}{x^{2}}\right)^{2} – 2 = 3^{2} – 2 = 9 – 2 = 7. \]

Hint 7

Substitute back into the expression from Hint 1: \[ x^{11} – 7x^{7} + x^{3} = x^{7}\!\Big(\underbrace{x^{4} + \frac{1}{x^{4}}}_{=7} – 7\Big) = x^{7}(7 – 7) = x^{7} \cdot 0 = 0. \]

Final Answer

(B) 0


AMC 12A — Problems

AMC 12A 2023 ( Problem 12 )

What is the value of
2^3-1^3+4^3-3^3+6^3-5^3+\cdots+18^3-17^3\ ?

  • 2023
  • 2679
  • 2941
  • 3159
  • 3235
Hints & Final Answer
Hint 1

Rewrite as -(1^3+2^3+\cdots+18^3)+2(2^3+4^3+6^3+\cdots+18^3).

Hint 2

Use Identity 3. Sum of Cubes: 1^3+2^3+\cdots+n^3=\left(\frac{n(n+1)}{2}\right)^2.

Hint 3

Note 2^3+4^3+6^3+\cdots+18^3=8(1^3+2^3+\cdots+9^3).

Final Answer

(D) 3159

AMC 12A 2023 ( Problem 10 )

Positive real numbers x and y satisfy y^3=x^2 and (y-x)^2=4y^2. What is x+y?

  • 12
  • 18
  • 24
  • 36
  • 42
Hints & Final Answer
Hint 1

y-x=\pm 2y

Hint 2

Case 1: y-x=2y\ \Rightarrow\ y=-x

Hint 3

In Case 1:
y^3=x^2\ \Rightarrow\ (-x)^3=x^2\ \Rightarrow\ -x^3=x^2\ \Rightarrow\ x=0,-1
But x>0 (and y>0) so Case 1 gives no solution.

Hint 4

Case 2: y-x=-2y\ \Rightarrow\ x=3y

Hint 5

In Case 2:
y^3=x^2\ \Rightarrow\ y^3=(3y)^2\ \Rightarrow\ y^3=9y^2
\Rightarrow\ y=0,9; with y>0 we get y=9\Rightarrow x=27.

Final Answer

(D) 36

AMC 12A 2023 ( Problem 23 )

How many ordered pairs of positive real numbers (a,b) satisfy the equation (1+2a)(2+2b)(2a+b)=32ab\ ?

  • 0
  • 1
  • 2
  • 3
  • an infinite number
Hints & Final Answer
Hint 1

Use 28. AM–GM Inequality.

Hint 2

1+2a\ \ge\ 2\sqrt{2a}

Hint 3

2+2b\ \ge\ 2\sqrt{4b}=4\sqrt{b}

Hint 4

2a+b\ \ge\ 2\sqrt{2ab}

Hint 5

By multiplying Hints 2, 3, and 4:
(1+2a)(2+2b)(2a+b)\ \ge\ 32ab

Hint 6

By using 28. AM–GM Inequality, equality holds if and only if
1=2a,\qquad 2=2b,\qquad 2a=b

Hint 7

a=\tfrac12,\quad b=1

Final Answer

(B) 1

AMC 12A 2022 ( Problem 21 )

Let \(P(x)=x^{2022}+x^{1011}+1.\) Which of the following polynomials is a factor of \(P(x)\)?

  • \(x^2-x+1\)
  • \(x^2+x+1\)
  • \(x^4+1\)
  • \(x^6-x^3+1\)
  • \(x^6+x^3+1\)
Hints & Final Answer
Hint 1

Use 32. Remainder of Polynomial Division.

Hint 2

Checking Choice (A):
\(x^2-x+1=0 \Rightarrow x^2=x-1\)
\(x^3=x^2\cdot x=(x-1)x=x^2-x=(x-1)-x=-1\)
\[ x^{2022}+x^{1011}+1=(x^3)^{674}+(x^3)^{337}+1 =(-1)^{674}+(-1)^{337}+1 =1\ne0 \]

Hint 3

Checking Choice (B):
\(x^2+x+1=0 \Rightarrow x^2=-x-1\)
\(x^3=-x^2-x=x+1-x=1\)
\[ x^{2022}+x^{1011}+1=(x^3)^{674}+(x^3)^{337}+1 =1+1+1=3\ne0 \]

Hint 4

Checking Choice (C):
\(x^4+1=0 \Rightarrow x^4=-1\)
\(x^{2022}=x^2(x^4)^{505}=-x^2,\quad x^{1011}=x^3(x^4)^{252}=x^3\)
\(x^{2022}+x^{1011}+1=-x^2+x^3+1\ne0\)

Hint 5

Checking Choice (D):
\(x^6-x^3+1=0 \Rightarrow x^6=x^3-1\)
\(x^9=x^6\cdot x^3=(x^3-1)x^3=x^6-x^3=(x^3-1)-x^3=-1\)
\[ x^{2022}+x^{1011}+1 =(x^9)^{224}x^6+(x^9)^{112}x^3+1 =(-1)^{224}x^6+(-1)^{112}x^3+1 =x^6+x^3+1 =2x^3\ne0 \]

Hint 6

Checking Choice (E):
\(x^6+x^3+1=0 \Rightarrow x^6=-x^3-1\)
\(x^9=x^6\cdot x^3=(-x^3-1)x^3=-x^6-x^3=-( -x^3-1)-x^3=1\)
\[ x^{2022}+x^{1011}+1 =(x^9)^{224}x^6+(x^9)^{112}x^3+1 =1^{224}x^6+1^{112}x^3+1 =x^6+x^3+1 =0 \]

Final Answer

(E) \(x^6+x^3+1\)

AMC 12A Fall 2021 ( Problem 12 )

What is the number of terms with rational coefficients among the 1001 terms in the expansion of \((x\sqrt[3]{2}+y\sqrt{3})^{1000}\)?

  • 0
  • 166
  • 167
  • 500
  • 501
Hints & Final Answer
Hint 1

Use 20. Binomial Theorem.

Hint 2

Each term has the form \[ (x\sqrt[3]{2})^k(\sqrt{3}y)^{1000-k}, \quad 0 \le k \le 1000. \]

Hint 3

For \((\sqrt[3]{2})^k\) to be rational, \(k\) must be a multiple of 3.

Hint 4

For \((\sqrt{3})^{1000-k}\) to be rational, \(1000-k\) must be even, so \(k\) must be even.

Hint 5

So \(k\) must be a multiple of 6.

Hint 6

\[ 0 \le k = 6q \le 1000 \Rightarrow 0 \le q \le 166. \] There are 167 possible \(q\).

Final Answer: (C) 167

AMC 12A Spring 2021 ( Problem 19 )

How many solutions does the equation \[ \sin\!\Big(\tfrac{\pi}{2}\cos x\Big) = \cos\!\Big(\tfrac{\pi}{2}\sin x\Big) \] have in the closed interval \([0,\pi]\)?

  • 0
  • 1
  • 2
  • 3
  • 4
Hints & Final Answer
Hint 1

By 37. Trigonometric Transformations: \[ \cos\!\Big(\tfrac{\pi}{2}\sin x\Big) =\sin\!\Big(\tfrac{\pi}{2}-\tfrac{\pi}{2}\sin x\Big). \]

Hint 2

Use 36. General Solutions of Trigonometric Equations.

Hint 3

Case 1: \[ \tfrac{\pi}{2}\cos x = \tfrac{\pi}{2}-\tfrac{\pi}{2}\sin x+2\pi k \quad\Rightarrow\quad \sin x+\cos x=1+4k. \] Using identities: \[ \sqrt2\,\sin\!\big(x+\tfrac{\pi}{4}\big)=1 \Rightarrow \sin\!\big(x+\tfrac{\pi}{4}\big)=\tfrac{\sqrt2}{2}=\sin\tfrac{\pi}{4}. \]

Hint 4

By general solutions: \[ x+\tfrac{\pi}{4}=\tfrac{\pi}{4}+2\pi q \text{ or } \tfrac{3\pi}{4}+2\pi q \Rightarrow x=0,\tfrac{\pi}{2}. \]

Hint 5

Case 2: \[ \tfrac{\pi}{2}\cos x =-\Big(\tfrac{\pi}{2}-\tfrac{\pi}{2}\sin x\Big)+2\pi k \quad\Rightarrow\quad \cos x-\sin x=1+4(k-1). \] The feasible value is \(\cos x-\sin x=1.\) Then \[ -\sqrt2\,\sin\!\big(x-\tfrac{\pi}{4}\big)=1 \Rightarrow \sin\!\big(x-\tfrac{\pi}{4}\big)=-\tfrac{\sqrt2}{2} =\sin\!\big(-\tfrac{\pi}{4}\big). \]

Hint 6

General solutions: \[ x-\tfrac{\pi}{4}=-\tfrac{\pi}{4}+2\pi q \text{ or } \tfrac{5\pi}{4}+2\pi q \Rightarrow x=0 \text{ in }[0,\pi]. \] So the solutions in \([0,\pi]\) are \(x=0,\tfrac{\pi}{2}\).

Final Answer: (C) 2

AMC 12A Fall 2021 ( Problem 19 )

Let \(x\) be the least real number greater than 1 such that \(\sin(x)=\sin(x^2)\), where the arguments are in degrees. What is \(x\) rounded up to the closest integer?

  • 10
  • 13
  • 14
  • 19
  • 20
Hints & Final Answer
Hint 1

Use 36. General Solutions of Trigonometric Equations.

Hint 2

\[ \sin x = \sin x^2 \Rightarrow x^2 = x + 360k \text{ or } x^2 = 180 – x + 360k, \quad k\in\mathbb{Z} \quad(\text{degrees}) \]

Hint 3

Case 1: \(x^2 = x + 360k\). \[ x^2 – x – 360k = 0 \Rightarrow x = \frac{1 \pm \sqrt{1 + 1440k}}{2}. \] Minimum \(x>1\) occurs at \(k=1\): \[ x = \frac{1 + \sqrt{1441}}{2} \approx \frac{1+38}{2} = 19.5. \]

Hint 4

Case 2: \(x^2 = 180 – x + 360k\). \[ x^2 + x – 360k – 180 = 0 \Rightarrow x = \frac{-1 \pm \sqrt{1440k + 721}}{2}. \] Minimum \(x>1\) is at \(k=0\): \[ x = \frac{-1 + \sqrt{721}}{2} \approx \frac{-1 + 27}{2} = 13. \] Round up: 13.
Final Answer: (B) 13

AMC 12B — Problems

AMC 12B 2023 ( Problem 14 )

For how many ordered pairs (a, b) of integers does the polynomial x^3 + ax^2 + bx + 6 have 3 distinct integer roots?

  • 5
  • 6
  • 8
  • 7
  • 4
Hints & Final Answer
Hint 1

Use 30. Vieta’s Formula.

Hint 2

r_1 + r_2 + r_3 = -a
r_1 r_2 + r_1 r_3 + r_2 r_3 = b
r_1 r_2 r_3 = -6

Hint 3

r_1 r_2 r_3 = -6
r_1, r_2, and r_3 are integer roots.
WLOG, assume r_1 < r_2 < r_3

Hint 4

Use casework:

r1r2r3ab
-116-6-1
-123-41
-213-2-5
-3120-7
-3-2-1611
Final Answer

(A) 5

AMC 12B 2022 ( Problem 4 )

For how many values of the constant \(k\) will the polynomial \(x^2 + kx + 36\) have two distinct integer roots?

  • 6
  • 8
  • 9
  • 14
  • 16
Hints & Final Answer
Hint 1

Use 30. Vieta’s Formula.

Hint 2

\[ r_1 + r_2 = -k,\quad r_1 r_2 = 36 \]

Hint 3

\(36 = 2^2\cdot 3^2\). Use 33. Number of Positive Divisors.

Hint 4

\(36\) has \((2+1)(2+1)=9\) positive divisors. Roots \(r_1\) and \(r_2\) are distinct, and \(k=-(r_1+r_2).\)

Hint 5

\(r_1 < r_2 \in \{\pm1,\pm2,\pm3,\pm4,\pm6,\pm9,\pm12,\pm18,\pm36\}\)
Pairs with product \(36\): \((1,36), (-1,-36), (2,18), (-2,-18), (3,12), (-3,-12), (4,9), (-4,-9)\)

Hint 6

\[ k = -(r_1 + r_2) = \pm 37,\ \pm 20,\ \pm 15,\ \pm 13 \]

Final Answer

(B) 8

AMC 12B 2021 Fall ( Problem 13 )

Let \(c=\dfrac{2\pi}{11}.\) What is the value of \[ \frac{\sin 3c \cdot \sin 6c \cdot \sin 9c \cdot \sin 12c \cdot \sin 15c} {\sin c \cdot \sin 2c \cdot \sin 3c \cdot \sin 4c \cdot \sin 5c}\ ? \]

  • \(-1\)
  • \(-\dfrac{\sqrt{11}}{5}\)
  • \(\dfrac{\sqrt{11}}{5}\)
  • \(\dfrac{10}{11}\)
  • 1
Hints & Final Answer
Hint 1

\(c=\frac{2\pi}{11}\Rightarrow 11c=2\pi.\)

Hint 2

Use 37. Trigonometry Transformations.

Hint 3

\(\sin 6c=\sin(2\pi-6c)=-\sin(11c-6c)=-\sin 5c.\)

Hint 4

\(\sin 9c=\sin(11c-2c)=-\sin 2c.\)

Hint 5

\(\sin 12c=\sin(11c+c)=\sin c.\)

Hint 6

\(\sin 15c=\sin(11c+4c)=\sin 4c.\)

Hint 7

\[ \frac{\sin3c\cdot\sin6c\cdot\sin9c\cdot\sin12c\cdot\sin15c} {\sin c\cdot\sin2c\cdot\sin3c\cdot\sin4c\cdot\sin5c} = \frac{\sin3c\cdot(-\sin5c)\cdot(-\sin2c)\cdot\sin c\cdot\sin4c} {\sin c\cdot\sin2c\cdot\sin3c\cdot\sin4c\cdot\sin5c} =1. \]

Final Answer

(E) 1

AMC 12B 2021 Spring ( Problem 16 )

Let \(g(x)\) be a polynomial with leading coefficient 1, whose three roots are the reciprocals of the three roots of \(f(x) = x^3 + ax^2 + bx + c,\) where \(1 < a < b < c.\) What is \(g(1)\) in terms of \(a, b,\) and \(c\)?

  • \(\dfrac{1 + a + b + c}{c}\)
  • \(1 + a + b + c\)
  • \(\dfrac{1 + a + b + c}{c^2}\)
  • \(\dfrac{a + b + c}{c^2}\)
  • \(\dfrac{1 + a + b + c}{a + b + c}\)
Hints & Final Answer
Hint 1

Use Vieta’s Formula.

Hint 2

If \(r_1, r_2, r_3\) are the roots of \(f(x)\), then \[ r_1 + r_2 + r_3 = -a,\quad r_1r_2 + r_2r_3 + r_3r_1 = b,\quad r_1r_2r_3 = -c. \]

Hint 3

The polynomial \(g(x)\) has reciprocal roots, so \[ g(x) = (x – \tfrac{1}{r_1})(x – \tfrac{1}{r_2})(x – \tfrac{1}{r_3}). \]

Hint 4

Evaluate \(g(1)\): \[ g(1) = (1 – \tfrac{1}{r_1})(1 – \tfrac{1}{r_2})(1 – \tfrac{1}{r_3}) = \frac{(r_1-1)(r_2-1)(r_3-1)}{r_1r_2r_3} \] \[ = \frac{r_1r_2r_3 – (r_1r_2 + r_2r_3 + r_3r_1) + (r_1+r_2+r_3) – 1}{r_1r_2r_3}. \] Substitute from Vieta: \[ g(1) = \frac{-c – b – a – 1}{-c} = \frac{1 + a + b + c}{c}. \]

Final Answer

(A) \(\dfrac{1 + a + b + c}{c}\)

AMC 12B 2021 Spring ( Problem 20 )

Let \(Q(z)\) and \(R(z)\) be the unique polynomials such that
z^{2021}+1=(z^2+z+1)\,Q(z)+R(z)
and the degree of \(R\) is less than \(2\). What is \(R(z)\)?

  • -z
  • -1
  • 2021
  • z+1
  • 2z+1
Hints & Final Answer
Hint 1

Use the Remainder Theorem for polynomial division modulo \(z^2+z+1\).

Hint 2

From \(z^2+z+1=0\) we get
z^2=-z-1 and hence
z^3=1 (for \(z\neq1\)).

Hint 3

Reduce the power:
z^{2021}=(z^3)^{673}\,z^2\equiv z^2\pmod{z^2+z+1}.
So
z^{2021}+1\equiv z^2+1\equiv(-z-1)+1=-z.
Therefore the remainder \(R(z)\) is \(-z\).

Final Answer

(A) -z


AMC 8 — Problems

AMC 8 2022 ( Problem 8 )

What is the value of
\frac{1}{3}\cdot\frac{2}{4}\cdot\frac{3}{5}\cdots\frac{18}{20}\cdot\frac{19}{21}\cdot\frac{20}{22} ?

  • \frac{1}{462}
  • \frac{1}{231}
  • \frac{1}{132}
  • \frac{2}{213}
  • \frac{1}{22}
Hints & Final Answer
Hint 1

Use Identity 5. Telescoping Series.

Final Answer

(E) \frac{1}{22}


AIME — Problems

AIME II 2022 ( Problem 10 )

Find the remainder when \binom{3}{2}+\binom{4}{2}+\cdots+\binom{40}{2} is divided by 1000.

Hints & Final Answer
Hint 1

\binom{\binom{k}{2}}{2} = \frac{\binom{k}{2}\Big(\binom{k}{2}-1\Big)}{2} = \frac{\frac{k(k-1)}{2}\cdot\frac{k(k-1)-2}{2}}{2} \\ = \frac{1}{8}\,k(k-1)\big(k^2-k-2\big) = \frac{1}{8}\,k(k-1)(k-2)(k+1) \\ = \frac{1}{8}\cdot 4!\,\binom{k+1}{4} = 3\binom{k+1}{4}

Hint 2

Use 16. Hockey Stick Identity

Hint 3

3\binom{42}{5}=\frac{42\times41\times40\times39\times38}{5\times4\times2} =42\times41\times39\times38=(40+2)(40+1)(40-1)(40-2)

Hint 4

Use 10. Difference of Squares

Hint 5

(1600-4)(1600-1)

Final Answer

004

USAJMO — Problems

USAJMO 2023 ( Problem 1 )

Find all triples of positive integers (x, y, z) that satisfy the equation 2(x + y + z + 2xyz)^2 = (2xy + 2yz + 2zx + 1)^2 + 2023.

Hints & Final Answer
Hint 1

Use 24. Squares of a sum.

Hint 2

Expand and simplify it.

Hint 3

8x^2y^2z^2 - 4x^2y^2 - 4y^2z^2 - 4z^2x^2 + 2x^2 + 2y^2 + 2z^2 - 1 = 2023

Hint 4

Use 26. Factorization of three binomials.

Hint 5

(2x^2 - 1)(2y^2 - 1)(2z^2 - 1) = 2023

Hint 6

Prime factorize 2023.

Hint 7

(2x^2 - 1)(2y^2 - 1)(2z^2 - 1) = 7 \times 17^2

Hint 8

When a \in \mathbb{Z}^+, then 2a^2 - 1 \in \{1, 7, 17, 119, 289, 2023\}

Hint 9

a^2 = 1, 4, 9, 60, 145, 1012
\Rightarrow a = 1, 2, 3, \times, \times, \times

Hint 10

2x^2 - 1, 2y^2 - 1, 2z^2 - 1 \in \{1, 7, 17\}

Hint 11

2023 = 7 \times 17 \times 17

Hint 12

2x^2 - 1, 2y^2 - 1, 2z^2 - 1 is a permutation of \{7, 17, 17\}

Final Answer

(x, y, z) = (2, 3, 3), (3, 2, 3), (3, 3, 2)


USAMO — Problems

USAMO 2018 ( Problem 1 )

Let a, b, c be positive real numbers such that a + b + c = 4\sqrt[3]{abc}. Prove that 2(ab + bc + ca) + 4\min(a^2, b^2, c^2) \ge a^2 + b^2 + c^2.

Hints & Final Answer
Hint 1

WLOG, assume a \le b \le c.

Hint 2

We should prove 2(ab + bc + ca) + 4a^2 \ge a^2 + b^2 + c^2.

Hint 3

Use 24. Square of a Sum.

Hint 4

Add 2ab + 2ac + 2bc to both sides.

Hint 5

We should prove 4(ab + bc + ca + a^2) \ge (a + b + c)^2.

Hint 6

Use a + b + c = 4\sqrt[3]{abc}.

Hint 7

We should prove 4(4a\sqrt[3]{abc} + bc) \ge 16\sqrt[3]{a^2b^2c^2}.

Hint 8

Use 28. AM–GM Inequality.

Hint 9

By using 28. AM–GM Inequality, \frac{4a\sqrt[3]{abc} + bc}{2} \ge \sqrt{4a\sqrt[3]{abc} \cdot bc} = 2\sqrt[3]{a^2b^2c^2}.

Final Answer

Multiply both sides of Hint 9 by 8 to get the desired result.


MathCounts — Problems

MathCounts 2025 Chapter — Sprint Round ( Problem 27 )

Let 2^n be the greatest power of 2 that divides the expression shown. What is the value of n?

1\times2\times3\times4 + 2\times3\times4\times5 + 3\times4\times5\times6 + \cdots + 25\times26\times27\times28

Hints & Final Answer
Hint 1

a(a+1)(a+2)(a+3) = 4!\binom{a+3}{4}

Hint 2

Use Identity 16. Hockey Stick Identity.

Final Answer

3

MathCounts 2025 State — Sprint Round ( Problem 30 )

If x and y are real numbers such that (4 - x)(4 + y) = 2 and (4 + x)(4 - y) = 3, what is the value of (x^2 - 1)(y^2 - 1)? Express your answer as a common fraction.

Hints & Final Answer
Hint 1

Use Identity 22. Approaches to Similar Equations.

Hint 2

Add the two equations and find the value of xy.

Hint 3

Substitute xy into the first equation and find x - y.

Hint 4

Square x - y and use Identity 7. Square of a Difference.

Final Answer

\frac{9999}{64}


HMMT — Problems

HMMT 2025 — Algebra & Number Theory Round ( Problem 7 )

There exists a unique triple (a, b, c) of positive real numbers that satisfies the equations 2(a^2 + 1) = 3(b^2 + 1) = 4(c^2 + 1) and ab + bc + ca = 1.
Compute a + b + c.

Hints & Final Answer
Hint 1

Use 27. Expansion (or factorization) of two binomials.

Hint 2

a^2 + 1 = a^2 + ab + ac + ca = (a + b)(a + c)

Hint 3

2(a + b)(a + c) = 3(b + a)(b + c) = 4(c + a)(c + b)

Hint 4

Let a + b = x,\ a + c = y,\ b + c = z.

Hint 5

2xy = 3yz = 4xz

Hint 6

y = \tfrac{3x}{4},\quad z = \tfrac{x}{2}

Hint 7

a + b = x,\ a + c = \tfrac{3x}{4},\ b + c = \tfrac{x}{2}

Hint 8

a = \tfrac{5x}{8},\quad b = \tfrac{3x}{8},\quad c = \tfrac{x}{8}

Hint 9

ab + ac + bc = 1

Hint 10

x = \tfrac{8}{\sqrt{23}}

Hint 11

a + b + c = \tfrac{x + y + z}{2}

Final Answer

\tfrac{9}{\sqrt{23}} = \tfrac{9\sqrt{23}}{23}


BMO — Problems

BMO1 2016/2017 ( Problem 3 )

Determine all pairs (m,n) of positive integers which satisfy the equation n^2 - 6n = m^2 + m - 10.

Hints & Final Answer
Hint 1

Use 6. Squares of a sum, 7. Squares of a difference and 10. Difference of squares

Hint 2

(n-3)^2 - 9 = (m+\tfrac{1}{2})^2 - \tfrac{1}{4} - 10

Hint 3

5 = (2m+1)^2 - (2n-6)^2 = (2m+2n-5)(2m-2n+7)

Hint 4

2m+2n-5 and 2m-2n+7 are factors of 5

Final Answer

(m,n) = (1,2),\ (1,4)


SMO — Problems

SMO 2025 Junior ( Problem 8 )

If x and y are positive integers such that
xy - 9x - 9y = 20 find the value of x^2 + y^2.

Hints & Final Answer
Hint 1

Use Identity 23: SFFT (Simon’s Favorite Factoring Trick).

Hint 2

(x-9)(y-9)=101

Hint 3

Since x and y are positive integers, \text{w.l.o.g. } x-9=1,\ y-9=101.

Final Answer

12200

SMO 2025 Senior ( Problem 20 )

The roots of the polynomial 10x^3 - 39x^2 + 29x - 6 are the height, length, and width of a rectangular box (right rectangular prism). A new rectangular box is formed by lengthening each edge of the original box by 2 units. What is the volume of the new box?

  • \dfrac{24}{5}
  • \dfrac{42}{5}
  • \dfrac{81}{5}
  • 30
  • 48
Hints & Final Answer
Hint 1

Use 30. Vieta’s Formula.

Hint 2

\text{Volume} = (r_1 + 2)(r_2 + 2)(r_3 + 2)

Hint 3

By using 30. Vieta’s Formula:
\begin{aligned} r_1 + r_2 + r_3 &= \frac{39}{10}, \\ r_1r_2 + r_1r_3 + r_2r_3 &= \frac{29}{10}, \\ r_1r_2r_3 &= \frac{6}{10}. \end{aligned}

Hint 4

Expand:
(r_1 + 2)(r_2 + 2)(r_3 + 2) = r_1r_2r_3 + 2(r_1r_2 + r_2r_3 + r_3r_1) + 4(r_1 + r_2 + r_3) + 8. Substitute from Vieta’s results:
\frac{6}{10} + 2\left(\frac{29}{10}\right) + 4\left(\frac{39}{10}\right) + 8 = 30.

Final Answer

(D) 30

SMO 2025 Open ( Problem 7 )

Let y = \sum_{k=0}^{20} \binom{20}{k}^2. Find the number of consecutive zeros at the end of the number (20!)^2 y when it is written in its decimal representation.

Hints & Final Answer
Hint 1

Use Identity 17. Vandermonde’s Identity.

Hint 2

y = \binom{40}{20}

Hint 3

(20!)^2 y = 40!

Hint 4

Find the number of factors of 5 in 40!.

Hint 5

Note that 25 contributes two factors of 5.

Final Answer

9